Put R under double integral












6















When I use $iint_R f(x,y)dA$ the letter $R$ is to the right of the double integral sign. How to make it under the sign? This is a simple question but I couldn't find a related question.










share|improve this question




















  • 2





    @JouleV I managed to mess up the markdown quoting:-), I'll delete and repost the comment, thanks.

    – David Carlisle
    Apr 19 at 8:46






  • 3





    you are using inline mathematics ($) the entire design of the layout for inline math is to make it fit within the normal line spacing of the text in a paragraph so limits move to subscript position, if you need the display style it it best to set it as a math display([...])

    – David Carlisle
    Apr 19 at 8:46
















6















When I use $iint_R f(x,y)dA$ the letter $R$ is to the right of the double integral sign. How to make it under the sign? This is a simple question but I couldn't find a related question.










share|improve this question




















  • 2





    @JouleV I managed to mess up the markdown quoting:-), I'll delete and repost the comment, thanks.

    – David Carlisle
    Apr 19 at 8:46






  • 3





    you are using inline mathematics ($) the entire design of the layout for inline math is to make it fit within the normal line spacing of the text in a paragraph so limits move to subscript position, if you need the display style it it best to set it as a math display([...])

    – David Carlisle
    Apr 19 at 8:46














6












6








6








When I use $iint_R f(x,y)dA$ the letter $R$ is to the right of the double integral sign. How to make it under the sign? This is a simple question but I couldn't find a related question.










share|improve this question
















When I use $iint_R f(x,y)dA$ the letter $R$ is to the right of the double integral sign. How to make it under the sign? This is a simple question but I couldn't find a related question.







subscripts






share|improve this question















share|improve this question













share|improve this question




share|improve this question








edited Apr 19 at 6:46









JouleV

15.3k22667




15.3k22667










asked Apr 19 at 6:46









Haoran ChenHaoran Chen

25918




25918








  • 2





    @JouleV I managed to mess up the markdown quoting:-), I'll delete and repost the comment, thanks.

    – David Carlisle
    Apr 19 at 8:46






  • 3





    you are using inline mathematics ($) the entire design of the layout for inline math is to make it fit within the normal line spacing of the text in a paragraph so limits move to subscript position, if you need the display style it it best to set it as a math display([...])

    – David Carlisle
    Apr 19 at 8:46














  • 2





    @JouleV I managed to mess up the markdown quoting:-), I'll delete and repost the comment, thanks.

    – David Carlisle
    Apr 19 at 8:46






  • 3





    you are using inline mathematics ($) the entire design of the layout for inline math is to make it fit within the normal line spacing of the text in a paragraph so limits move to subscript position, if you need the display style it it best to set it as a math display([...])

    – David Carlisle
    Apr 19 at 8:46








2




2





@JouleV I managed to mess up the markdown quoting:-), I'll delete and repost the comment, thanks.

– David Carlisle
Apr 19 at 8:46





@JouleV I managed to mess up the markdown quoting:-), I'll delete and repost the comment, thanks.

– David Carlisle
Apr 19 at 8:46




3




3





you are using inline mathematics ($) the entire design of the layout for inline math is to make it fit within the normal line spacing of the text in a paragraph so limits move to subscript position, if you need the display style it it best to set it as a math display([...])

– David Carlisle
Apr 19 at 8:46





you are using inline mathematics ($) the entire design of the layout for inline math is to make it fit within the normal line spacing of the text in a paragraph so limits move to subscript position, if you need the display style it it best to set it as a math display([...])

– David Carlisle
Apr 19 at 8:46










2 Answers
2






active

oldest

votes


















13














Apart from JouleV's nice answer, you can use limits option to typeset the inline with equation with limits under the integral symbol.



documentclass{article}
usepackage{amsmath}
begin{document}
$iintlimits_a f(x,y) dA$
end{document}


to get:



enter image description here






share|improve this answer





















  • 3





    I'd say that iintlimits_R centers the R term far better than underset{R}{iint} does.

    – Mico
    Apr 19 at 8:58








  • 1





    No upper limit should be used when limits follows a command for multiple integrals. You should also respect the OP’s preference for the differential.

    – egreg
    Apr 19 at 9:09











  • @egreg I have updated my answer, thanks for the remark. Could you also briefly explain why no upper limit must be used?

    – Raaja
    Apr 19 at 9:13






  • 2





    @Raaja Because they’re badly positioned.

    – egreg
    Apr 19 at 9:14











  • @egreg Huhh, like that ;) thanks.

    – Raaja
    Apr 19 at 9:15



















4














I don't think this is a good idea, but if you want to have it, you can use underset:



documentclass{standalone}
usepackage{amsmath}
begin{document}
$underset{R}{iint} f(x,y)dA$
end{document}


enter image description here






share|improve this answer



















  • 1





    Sorry, but this is not the best choice.

    – egreg
    Apr 19 at 9:10






  • 1





    @egreg Yes, it may not be the best, but it surely is a bad one. I will never write like that in my documents

    – JouleV
    Apr 19 at 9:19














Your Answer








StackExchange.ready(function() {
var channelOptions = {
tags: "".split(" "),
id: "85"
};
initTagRenderer("".split(" "), "".split(" "), channelOptions);

StackExchange.using("externalEditor", function() {
// Have to fire editor after snippets, if snippets enabled
if (StackExchange.settings.snippets.snippetsEnabled) {
StackExchange.using("snippets", function() {
createEditor();
});
}
else {
createEditor();
}
});

function createEditor() {
StackExchange.prepareEditor({
heartbeatType: 'answer',
autoActivateHeartbeat: false,
convertImagesToLinks: false,
noModals: true,
showLowRepImageUploadWarning: true,
reputationToPostImages: null,
bindNavPrevention: true,
postfix: "",
imageUploader: {
brandingHtml: "Powered by u003ca class="icon-imgur-white" href="https://imgur.com/"u003eu003c/au003e",
contentPolicyHtml: "User contributions licensed under u003ca href="https://creativecommons.org/licenses/by-sa/3.0/"u003ecc by-sa 3.0 with attribution requiredu003c/au003e u003ca href="https://stackoverflow.com/legal/content-policy"u003e(content policy)u003c/au003e",
allowUrls: true
},
onDemand: true,
discardSelector: ".discard-answer"
,immediatelyShowMarkdownHelp:true
});


}
});














draft saved

draft discarded


















StackExchange.ready(
function () {
StackExchange.openid.initPostLogin('.new-post-login', 'https%3a%2f%2ftex.stackexchange.com%2fquestions%2f485553%2fput-r-under-double-integral%23new-answer', 'question_page');
}
);

Post as a guest















Required, but never shown

























2 Answers
2






active

oldest

votes








2 Answers
2






active

oldest

votes









active

oldest

votes






active

oldest

votes









13














Apart from JouleV's nice answer, you can use limits option to typeset the inline with equation with limits under the integral symbol.



documentclass{article}
usepackage{amsmath}
begin{document}
$iintlimits_a f(x,y) dA$
end{document}


to get:



enter image description here






share|improve this answer





















  • 3





    I'd say that iintlimits_R centers the R term far better than underset{R}{iint} does.

    – Mico
    Apr 19 at 8:58








  • 1





    No upper limit should be used when limits follows a command for multiple integrals. You should also respect the OP’s preference for the differential.

    – egreg
    Apr 19 at 9:09











  • @egreg I have updated my answer, thanks for the remark. Could you also briefly explain why no upper limit must be used?

    – Raaja
    Apr 19 at 9:13






  • 2





    @Raaja Because they’re badly positioned.

    – egreg
    Apr 19 at 9:14











  • @egreg Huhh, like that ;) thanks.

    – Raaja
    Apr 19 at 9:15
















13














Apart from JouleV's nice answer, you can use limits option to typeset the inline with equation with limits under the integral symbol.



documentclass{article}
usepackage{amsmath}
begin{document}
$iintlimits_a f(x,y) dA$
end{document}


to get:



enter image description here






share|improve this answer





















  • 3





    I'd say that iintlimits_R centers the R term far better than underset{R}{iint} does.

    – Mico
    Apr 19 at 8:58








  • 1





    No upper limit should be used when limits follows a command for multiple integrals. You should also respect the OP’s preference for the differential.

    – egreg
    Apr 19 at 9:09











  • @egreg I have updated my answer, thanks for the remark. Could you also briefly explain why no upper limit must be used?

    – Raaja
    Apr 19 at 9:13






  • 2





    @Raaja Because they’re badly positioned.

    – egreg
    Apr 19 at 9:14











  • @egreg Huhh, like that ;) thanks.

    – Raaja
    Apr 19 at 9:15














13












13








13







Apart from JouleV's nice answer, you can use limits option to typeset the inline with equation with limits under the integral symbol.



documentclass{article}
usepackage{amsmath}
begin{document}
$iintlimits_a f(x,y) dA$
end{document}


to get:



enter image description here






share|improve this answer















Apart from JouleV's nice answer, you can use limits option to typeset the inline with equation with limits under the integral symbol.



documentclass{article}
usepackage{amsmath}
begin{document}
$iintlimits_a f(x,y) dA$
end{document}


to get:



enter image description here







share|improve this answer














share|improve this answer



share|improve this answer








edited Apr 19 at 9:12

























answered Apr 19 at 6:53









RaajaRaaja

5,46121645




5,46121645








  • 3





    I'd say that iintlimits_R centers the R term far better than underset{R}{iint} does.

    – Mico
    Apr 19 at 8:58








  • 1





    No upper limit should be used when limits follows a command for multiple integrals. You should also respect the OP’s preference for the differential.

    – egreg
    Apr 19 at 9:09











  • @egreg I have updated my answer, thanks for the remark. Could you also briefly explain why no upper limit must be used?

    – Raaja
    Apr 19 at 9:13






  • 2





    @Raaja Because they’re badly positioned.

    – egreg
    Apr 19 at 9:14











  • @egreg Huhh, like that ;) thanks.

    – Raaja
    Apr 19 at 9:15














  • 3





    I'd say that iintlimits_R centers the R term far better than underset{R}{iint} does.

    – Mico
    Apr 19 at 8:58








  • 1





    No upper limit should be used when limits follows a command for multiple integrals. You should also respect the OP’s preference for the differential.

    – egreg
    Apr 19 at 9:09











  • @egreg I have updated my answer, thanks for the remark. Could you also briefly explain why no upper limit must be used?

    – Raaja
    Apr 19 at 9:13






  • 2





    @Raaja Because they’re badly positioned.

    – egreg
    Apr 19 at 9:14











  • @egreg Huhh, like that ;) thanks.

    – Raaja
    Apr 19 at 9:15








3




3





I'd say that iintlimits_R centers the R term far better than underset{R}{iint} does.

– Mico
Apr 19 at 8:58







I'd say that iintlimits_R centers the R term far better than underset{R}{iint} does.

– Mico
Apr 19 at 8:58






1




1





No upper limit should be used when limits follows a command for multiple integrals. You should also respect the OP’s preference for the differential.

– egreg
Apr 19 at 9:09





No upper limit should be used when limits follows a command for multiple integrals. You should also respect the OP’s preference for the differential.

– egreg
Apr 19 at 9:09













@egreg I have updated my answer, thanks for the remark. Could you also briefly explain why no upper limit must be used?

– Raaja
Apr 19 at 9:13





@egreg I have updated my answer, thanks for the remark. Could you also briefly explain why no upper limit must be used?

– Raaja
Apr 19 at 9:13




2




2





@Raaja Because they’re badly positioned.

– egreg
Apr 19 at 9:14





@Raaja Because they’re badly positioned.

– egreg
Apr 19 at 9:14













@egreg Huhh, like that ;) thanks.

– Raaja
Apr 19 at 9:15





@egreg Huhh, like that ;) thanks.

– Raaja
Apr 19 at 9:15











4














I don't think this is a good idea, but if you want to have it, you can use underset:



documentclass{standalone}
usepackage{amsmath}
begin{document}
$underset{R}{iint} f(x,y)dA$
end{document}


enter image description here






share|improve this answer



















  • 1





    Sorry, but this is not the best choice.

    – egreg
    Apr 19 at 9:10






  • 1





    @egreg Yes, it may not be the best, but it surely is a bad one. I will never write like that in my documents

    – JouleV
    Apr 19 at 9:19


















4














I don't think this is a good idea, but if you want to have it, you can use underset:



documentclass{standalone}
usepackage{amsmath}
begin{document}
$underset{R}{iint} f(x,y)dA$
end{document}


enter image description here






share|improve this answer



















  • 1





    Sorry, but this is not the best choice.

    – egreg
    Apr 19 at 9:10






  • 1





    @egreg Yes, it may not be the best, but it surely is a bad one. I will never write like that in my documents

    – JouleV
    Apr 19 at 9:19
















4












4








4







I don't think this is a good idea, but if you want to have it, you can use underset:



documentclass{standalone}
usepackage{amsmath}
begin{document}
$underset{R}{iint} f(x,y)dA$
end{document}


enter image description here






share|improve this answer













I don't think this is a good idea, but if you want to have it, you can use underset:



documentclass{standalone}
usepackage{amsmath}
begin{document}
$underset{R}{iint} f(x,y)dA$
end{document}


enter image description here







share|improve this answer












share|improve this answer



share|improve this answer










answered Apr 19 at 6:50









JouleVJouleV

15.3k22667




15.3k22667








  • 1





    Sorry, but this is not the best choice.

    – egreg
    Apr 19 at 9:10






  • 1





    @egreg Yes, it may not be the best, but it surely is a bad one. I will never write like that in my documents

    – JouleV
    Apr 19 at 9:19
















  • 1





    Sorry, but this is not the best choice.

    – egreg
    Apr 19 at 9:10






  • 1





    @egreg Yes, it may not be the best, but it surely is a bad one. I will never write like that in my documents

    – JouleV
    Apr 19 at 9:19










1




1





Sorry, but this is not the best choice.

– egreg
Apr 19 at 9:10





Sorry, but this is not the best choice.

– egreg
Apr 19 at 9:10




1




1





@egreg Yes, it may not be the best, but it surely is a bad one. I will never write like that in my documents

– JouleV
Apr 19 at 9:19







@egreg Yes, it may not be the best, but it surely is a bad one. I will never write like that in my documents

– JouleV
Apr 19 at 9:19




















draft saved

draft discarded




















































Thanks for contributing an answer to TeX - LaTeX Stack Exchange!


  • Please be sure to answer the question. Provide details and share your research!

But avoid



  • Asking for help, clarification, or responding to other answers.

  • Making statements based on opinion; back them up with references or personal experience.


To learn more, see our tips on writing great answers.




draft saved


draft discarded














StackExchange.ready(
function () {
StackExchange.openid.initPostLogin('.new-post-login', 'https%3a%2f%2ftex.stackexchange.com%2fquestions%2f485553%2fput-r-under-double-integral%23new-answer', 'question_page');
}
);

Post as a guest















Required, but never shown





















































Required, but never shown














Required, but never shown












Required, but never shown







Required, but never shown

































Required, but never shown














Required, but never shown












Required, but never shown







Required, but never shown







Popular posts from this blog

Plaza Victoria

In PowerPoint, is there a keyboard shortcut for bulleted / numbered list?

How to put 3 figures in Latex with 2 figures side by side and 1 below these side by side images but in...